Difference between revisions of "2023 AMC 10A Problems/Problem 16"

(Solution 1 (3 min solve))
(Solution 2)
Line 11: Line 11:
  
 
==Solution 2==
 
==Solution 2==
First, every player played the other, so there's <math>n\c2</math> games. Also, if the right-handed won <math>x</math> games, the left handed won <math>7/5x</math>, meaning that the total amount of games was <math>12/5x</math>, so the total amount of games is divisible by <math>12</math>.  
+
First, every player played the other, so there's <math>n\choose2</math> games. Also, if the right-handed won <math>x</math> games, the left handed won <math>7/5x</math>, meaning that the total amount of games was <math>12/5x</math>, so the total amount of games is divisible by <math>12</math>.  
  
 
Then, we do something funny and look at the answer choices. Only <math>\boxed{B}</math> satisfies our 2 findings.
 
Then, we do something funny and look at the answer choices. Only <math>\boxed{B}</math> satisfies our 2 findings.

Revision as of 21:39, 9 November 2023

Problem

In a table tennis tournament every participant played every other participant exactly once. Although there were twice as many right-handed players as left-handed players, the number of games won by left-handed players was $40\%$ more than the number of games won by right-handed players. (There were no ties and no ambidextrous players.) What is the total number of games played?

$\textbf{(A) }15\qquad\textbf{(B) }36\qquad\textbf{(C) }45\qquad\textbf{(D) }48\qquad\textbf{(E) }66$

Solution 1 (3 min solve)

We know that the total amount of games must be the sum of games won by left and right handed players. Then, we can write $g = l + r$, and since $l = 1.4r$, $g = 2.4r$. Given that $r$ and $g$ are both integers, $g/2.4$ also must be an integer. From here we can see that $g$ must be divisible by 12, leaving only answers B and D. Now we know the formula for how many games are played in this tournament is $n(n-1)/2$, the sum of the first $n-1$ triangular numbers. Now setting 36 and 48 equal to the equation will show that two consecutive numbers must equal 72 or 96. Clearly $72=8*9$, so the answer is $\boxed{(B)}$.

~~ Antifreeze5420

Solution 2

First, every player played the other, so there's $n\choose2$ games. Also, if the right-handed won $x$ games, the left handed won $7/5x$, meaning that the total amount of games was $12/5x$, so the total amount of games is divisible by $12$.

Then, we do something funny and look at the answer choices. Only $\boxed{B}$ satisfies our 2 findings.

Video Solution 1 by OmegaLearn

https://youtu.be/BXgQIV2WbOA

See Also

2023 AMC 10A (ProblemsAnswer KeyResources)
Preceded by
Problem 15
Followed by
Problem 17
1 2 3 4 5 6 7 8 9 10 11 12 13 14 15 16 17 18 19 20 21 22 23 24 25
All AMC 10 Problems and Solutions

The problems on this page are copyrighted by the Mathematical Association of America's American Mathematics Competitions. AMC logo.png